Experimentelles Ergebnis kann nicht durch Theorie für 2-Feder-1-Masse-System erklärt werden

Wir haben 2 Feder-1-Masse-System in 2D wie gezeigt,

System

Hier mein kurzer Lösungsversuch:

F X = F 1 X + F 2 X = k ( X + l ) ich ^ k ( X l ) ich ^ = 2 k X ich ^ X ¨ + 2 k M X = 0 ,
F j = F 1 j + F 2 j = 2 k j ȷ ^ j ¨ + 2 k M j = 0.
Allgemeine Lösungen für diese Gleichungen sind:
X ( T ) = A Sünde ( ω T ) + B cos ( ω T ) ,
j ( T ) = C Sünde ( ω T ) + D cos ( ω T ) ,
Wo ω = 2 k M . Bewertung der Anfangsbedingungen wie folgt;
X ( 0 ) = X 0 X ( 0 ) = A Sünde ( 0 ) + B cos ( 0 ) = B = X 0 ,
j ( 0 ) = j 0 j ( 0 ) = C Sünde ( 0 ) + D cos ( 0 ) = D = j 0 ,
X ˙ ( 0 ) = v 0 X = 0 X ˙ ( 0 ) = A ω cos ( 0 ) X 0 ω Sünde ( 0 ) = A = 0 ,
j ˙ ( 0 ) = v 0 j = 0 j ˙ ( 0 ) = C ω cos ( 0 ) j 0 ω Sünde ( 0 ) = C = 0 ,
X ( T ) = X 0 cos ( ω T ) , j ( T ) = j 0 cos ( ω T ) .
Ich habe diese Lösung mit einer anderen Methode überprüft, die hier in der ersten Antwort angegeben ist, und sie sind konsistent. Beachten Sie, dass in der letzten Gleichung ein kleiner Fehler ist, das sollte es sein M anstatt 2 M ; Sie können dies hier in der ersten Antwort überprüfen.

Ich habe eine Figur dieser Lösung gemacht und hier ist sie:

Geben Sie hier die Bildbeschreibung ein

In der Fakultät haben wir dieses Experiment durchgeführt und das Ergebnis sieht in etwa so aus (ebenfalls von mir gemacht):

Geben Sie hier die Bildbeschreibung ein

Die Punkte zeigen die Position der Masse. Der einzige Unterschied zwischen diesen beiden Bildern ist die Phasenverschiebung. Um die experimentelle Figur zu erhalten, füge ich eine Phasenverschiebung von zu hinzu j ( T ) Und;

j ( T ) = j 0 cos ( ω T + ϕ ) , ϕ = arctan ( j 0 / X 0 ) .

Und da ist auch noch folgendes: Als wir dieses Experiment im Labor durchführten, sagte der Ausbilder, dass die X ( T ) Und j ( T ) sollte eine Phasenverschiebung von haben π / 2 , wrt einander, was bedeutet, wenn X ( T ) cos ( ω T ) Dann j Sünde ( ω T ) und umgekehrt. Und das war der eigentliche Fall im Labor.

Meine Frage ist, wie kann ich diese Phasenverschiebung aus den Gleichungen - legal - gewinnen? Oder gibt es eine Erklärung?

Bearbeiten:

es ist ein 50 C M × 50 C M X j horizontale Ebene, also wird kein g auf das System angewendet. M = 570 G R Und k 60000 D j N C M . Ruhelänge der Federn ist l 0 = 13 C M . Zur Durchführung des Versuchs spannen wir zunächst beide Federn und befestigen sie an der Masse. Ein neues Gleichgewicht tritt ein, wenn die Länge der Federn ungefähr ist 25 C M . Ich denke, das ist eine ziemlich große Dehnung, aber soweit ich weiß, ist die Elastizität nicht gebrochen.

Hier ist ein kurzes Filmmaterial von Normalmoden und kleinen Schwingungen: https://www.youtube.com/watch?v=eyEpFeZO9W8 Im Labor haben wir dieses Experiment mit viel größeren Amplituden in beide Richtungen eingestellt. Ich werde so schnell wie möglich einige echte Fotos und Daten zur Verfügung stellen.

+1 für Mühe und Forschung. Es gibt keine Schwerkraft in Ihrer Analyse. Waren die Schwingungen auf eine horizontale Ebene beschränkt? Oder waren die Federn so straff, dass die Schwerkraft vernachlässigt werden kann?
Was steht auf den Achsen in Ihren Plots? Sind sie y(t) vs. x(t)?
Die Anfangsbedingungen bestimmen, ob die Oszillation für die X Und j Achsen gleichphasig sein oder nicht. Ich denke, das Experiment und die Theorie verwenden unterschiedliche Anfangsbedingungen.
Können Sie bitte weitere Details zu Ihrem Experiment angeben: Was sind die Ruhelänge und die Gleichgewichtslänge jeder Feder? Was sind die Werte von k Und M ? Ist der j Richtung vertikal, oder ist die X j Ebene horizontal? Wie bringt man das System zum Schwingen? Was sind Ihre tatsächlichen Ergebnisse? (Ich nehme an, dass Ihr letztes Diagramm eine Illustration und keine tatsächlichen Daten ist.)
es ist ein 50 C M × 50 C M X j horizontale Ebene, also wird kein g auf das System angewendet. M = 570 G R Und k 60000 D j N C M . Ruhelänge der Federn ist l 0 = 13 C M . Und ja, ich habe die Plots in dieser Frage gemacht. Die Achsen sind X j . Aber ich kann morgen nach dem Unterricht echte Daten hinzufügen.
Ja, Ihre tatsächlichen Daten wären sehr nützlich. Hast du ein Video von den Schwingungen gemacht?
Ich habe ein gutes Video der normalen Modi des Systems und eine wirklich kleine Oszillation, ich werde versuchen, dieses Video hier in nur einer Minute hinzuzufügen. Und ich werde morgen die aktuellen Daten liefern, auch das vollständige Video - wenn ich hier ein Video hinzufügen kann -
@Saba-Gibt es einen Unterschied zwischen dem ersten Dehnen der Federn und dem anschließenden Verbinden mit der Masse und dem Dehnen der Federn mit bereits angebrachter Masse?
@Saba Können Sie Daten aus Ihrem Experiment bereitstellen? zB Tabellen von x(t) und y(t). Hast du auch Daten von deiner Messung der Federkonstante?

Antworten (3)

Deine Bewegungsgleichungen sind falsch. Um zu verstehen, warum, betrachten Sie den Fall in diesem Bild:

Geben Sie hier die Bildbeschreibung ein

Was sind die X Und j Bestandteile der Kraft F Auf die Masse wirken?

Wenn die Ruhelänge der Feder ist l 0 und seine elastische Konstante ist k , die Kraft F Ist

F = k R ^ ( l l 0 ) = k R ^ ( X 2 + j 2 l 0 )

Wo R ^ wirkt in Richtung des roten Pfeils, dh R ^ = ( cos θ , Sünde θ ) . Der X Und j Komponenten sind daher

F X = k cos θ ( X 2 + j 2 l 0 ) F j = k Sünde θ ( X 2 + j 2 l 0 )

Wo

θ = arctan ( j X )

Wenn wir einer ähnlichen Methode wie Ihrer folgen würden, würden wir erhalten

F X = k ( X l 0 ) F j = k j

Was falsch ist und dem Fall zweier unabhängiger Federn mit identischen Konstanten entspricht, die auf die Masse einwirken.

Nehmen wir den Fall mit zwei identischen Federn:

Geben Sie hier die Bildbeschreibung ein

Basierend auf der vorherigen Analyse können Sie das leicht erkennen

F 1 = k R ^ 1 ( X 2 + j 2 l 0 ) F 2 = k R ^ 2 ( ( X L ) 2 + j 2 l 0 )

Wo R ^ 1 = ( cos θ 1 , Sünde θ 1 ) Und R ^ 2 = ( cos θ 2 , Sünde θ 2 ) . Daraus folgt, dass die X , j Komponenten sind

F X = k cos θ 1 ( X 2 + j 2 l 0 ) + k cos θ 2 ( ( X L ) 2 + j 2 l 0 ) F j = k Sünde θ 1 ( X 2 + j 2 l 0 ) k Sünde θ 2 ( ( X L ) 2 + j 2 l 0 )

Wo

θ 1 = arctan ( j X ) θ 2 = arctan ( L X j )

Die Bewegungsgleichungen sind daher recht kompliziert exakt zu lösen. Wenn Sie codieren können, würde ich vorschlagen, sie mit einem Integrator wie Velocity Verlet zu lösen .

Genau das hat Floris in diesem Beitrag getan: physical.stackexchange.com/questions/231364/… . Für kleine Winkel entspricht dies ungefähr Fx=-2kx und Fy=-2T/L. Wenn Sie es aufschreiben, werden Sie feststellen, dass die vernachlässigten Terme höherer Ordnung für eine hohe Zugkraft T sehr klein sind, selbst bei relativ großen Verschiebungen, wie z. B. x=y=L/2.
@rickboender Hatte das nicht gesehen, danke für den Hinweis. Ich kann die aktuelle Antwort um eine Diskussion über kleine Amplituden erweitern. Wie auch immer, die Frage von OP bezieht sich nicht speziell auf kleine Amplituden (auch wenn das Video den Fall kleiner Amplituden zeigt).
@rickboender Es wäre interessant zu sehen, ob eine kleine Amplitudenanalyse die Phasenverschiebung ergeben kann, die OP sagt, wir sollten bekommen, aber ich vermute nicht ...
Ich glaube auch nicht, dass die vom OP erwartete Phasenverschiebung angezeigt werden kann, die Frequenzen sind unterschiedlich, also gibt es nicht wirklich eine Phasenverschiebung. Was die kleinen Verschiebungen betrifft, ist dies normalerweise die einzige Möglichkeit, diese Art von Problemen beherrschbar zu halten. In nichtlinearen Systemen hängen Frequenz und Dämpfung von der Belastung ab, ändern sich also ständig und Sie benötigen genaue Messgeräte, um die gesamte Bewegung zu protokollieren.

Die Berechnung der Steifigkeit in j Richtung ist falsch. Die Steifheit drin j -Richtung hängt nicht von der Federsteifigkeit ab k , aber nur auf die Spannkraft T in den Federn und die Länge der Federn im Ruhezustand (wie sie an der Masse befestigt sind).

Die Steifheit drin j -Richtung ist gegeben durch:

F = T Sünde θ = T   j L

Wo θ ist der Winkel zwischen der Feder und den X-Achsen. Beachten Sie, dass das letzte „=“-Zeichen nur für kleine Verschiebungen gültig ist, aber das gilt auch für die gesamte Analyse. Wenn θ erhöht, die Wirkung von T nimmt mit ab cos θ , und der Einfluss der Federsteifigkeit nimmt mit zu Sünde θ .

Normalerweise gibt es keine Phasenverschiebung, weil die Frequenzen in X Und j Richtungen sind unterschiedlich. Sie können nur für bestimmte Werte von übereinstimmen k , Länge L des Frühlings u T . Wenn sie übereinstimmen, kann die Verschiebung wie Sie mit den Anfangsbedingungen bestimmt werden.

Wenn die Federn gespannt sind T im Gleichgewicht und die Amplituden klein sind, dann sind es die Rückstellkräfte F X 2 k X Und F j 2 T L j , Wo L ist die gedehnte Länge der Feder, wie in Querschwingung in Systemen mit 1 Masse und 2 Federn verstehen erläutert . Wenn die natürliche Länge der Federn ist L 0 Dann T = k ( L L 0 ) So F j 2 k ( 1 L 0 L ) . Die Schwingungen in der X Und j Richtungen sind ca. linear und unabhängig, also einfach harmonisch, aber die Frequenzen unterscheiden sich im Verhältnis F j / F X 1 L 0 L .

Dieser Frequenzunterschied bedeutet, dass die Phasendifferenz zwischen den X Und j Schwingungen nehmen allmählich zu. Die Bewegung ist nicht wie Ihre Graphen, die eine konstante Phasendifferenz zeigen. Stattdessen wechselt die Bewegung von einer linearen Schwingung wie in Graph 1 zu einer elliptischen Schwingung in Graph 2, die kreisförmig wird. Sie wird dann wieder elliptisch, diesmal aber mit der Linie (=Achse der Ellipse) an der y-Achse gespiegelt. Nachdem die Richtung der Schwingung wieder linear geworden ist, kehrt sie sich um und der Zyklus beginnt von neuem. Diese Bewegung wird durch eine Animation in Warum verhält sich die Vibration in meinem Draht so seltsam? und ist auch in deinem Video zu sehen.

Verwendung der von Ihnen bereitgestellten Daten ( L 0 = 13 C M , L = 25 C M ) , Dann 1 L 0 L 0,48 . Die Frequenzen sollten also im Verhältnis stehen F j / F X 0,69 . Von den 2 Läufen in der 1. Hälfte deines Videos, innerhalb von ca. 8s sind das 11 Zyklen X Oszillation und 7 Zyklen der j Schwingung, also F j / F X = 7 / 11 0,64 , was ziemlich nahe an der Vorhersage liegt.

Allerdings ist die X Und j Schwingungen scheinen nicht unabhängig voneinander zu sein. In der 2 läuft in der 2. Hälfte das Video, in dem X Und j Bewegungen treten gleichzeitig auf, das Verhältnis F j / F X ist ca. 8 / 9 anstatt 7 / 11 wenn diese Bewegungen getrennt sind. Der Frequenzunterschied ist deutlich kleiner und beide haben sich aufeinander zu bewegt. Dafür gibt es zwei Gründe: (i) Die Näherung für kleine Amplituden gilt nicht, also F X , F j jeweils abhängen X Und j - dh sie sind gekoppelt; (ii) Energie wird auch durch Reibung oder Hysterese gekoppelt. (Für ein Beispiel der Reibungskopplung zweier ansonsten unabhängiger Bewegungen siehe Rotationsphysik einer Spielkarte ).

Es ist nicht ersichtlich, wie die π / 2 eine von Ihrem Lehrer vorgeschlagene Phasendifferenz auftreten. Wenn eine Schwingung die andere antreibe, würde sie vorbeiziehen π / 2 . Das könnte passieren, wenn es zwei gekoppelte Massen gäbe, von denen eine viel schwerer als die andere wäre. Aber hier sind die Massen gleich.

Aus dem Video, F X 11 / 8 1.4 H z . Aus Ihren Messungen, vorausgesetzt k bezieht sich dann auf eine der beiden Federn F X 1 2 π 2 k M = 1 2 π 2 × 60 , 000 570 2.3 H z . Möglicherweise Ihr Wert von k das ist nicht richtig?


Die Gleichung, für die Sie hergeleitet haben F j gilt nur wann L L 0 . Dann F j / F X 1 Die Phasendifferenz bleibt also ca. Konstante. Wenn Sie das System aus dem Ruhezustand starten, ist die Phasendifferenz Null (Diagramm 1), da beide X Und j Beginnen Sie bei maximaler Verschiebung, damit sie die gleiche Phase haben. Die Amplituden müssen nicht gleich sein, da die Frequenz unabhängig von der Amplitude ist. Um eine konstante Phasendifferenz wie in Diagramm 2 zu haben, können Sie die Masse in die drücken X oder j Richtung, wenn Sie es loslassen.

Wenn die Amplitude der Schwingungen "groß" wird, dann die X Und j Schwingungen werden nichtlinear und gekoppelt. Sind die Federn im Gleichgewichtszustand sehr wenig oder gar nicht gespannt, sind die Querschwingungen auch bei kleinen Amplituden nichtlinear, mit einer Rückstellkraft proportional zu j 3 .